Difference between revisions of "1994 AJHSME Problems/Problem 1"

Line 5: Line 5:
  
 
==Solution==
 
==Solution==
C and D are the only answer choices where the numerator is close to half of the denominator. <math>\dfrac{3}{8}</math> is equal to .375 and <math>\dfrac{5}{12}</math> is equal to approximately .41. <math>\boxed{\text{D}}</math>
+
C and D are the only answer choices where the numerator is close to half of the denominator. <math>\dfrac{3}{8} = .375</math> and <math>\dfrac{5}{12} \approx .41</math>. Thus the answer is <math>\boxed{\text{(D)}\ \frac{5}{12}}</math>
 +
 
 +
==See Also==
 +
{{AJHSME box|year=1994|before=First <br /> Problem|num-a=2}}

Revision as of 00:09, 23 December 2012

Problem

Which of the following is the largest?

$\text{(A)}\ \dfrac{1}{3} \qquad \text{(B)}\ \dfrac{1}{4} \qquad \text{(C)}\ \dfrac{3}{8} \qquad \text{(D)}\ \dfrac{5}{12} \qquad \text{(E)}\ \dfrac{7}{24}$

Solution

C and D are the only answer choices where the numerator is close to half of the denominator. $\dfrac{3}{8} = .375$ and $\dfrac{5}{12} \approx .41$. Thus the answer is $\boxed{\text{(D)}\ \frac{5}{12}}$

See Also

1994 AJHSME (ProblemsAnswer KeyResources)
Preceded by
First
Problem
Followed by
Problem 2
1 2 3 4 5 6 7 8 9 10 11 12 13 14 15 16 17 18 19 20 21 22 23 24 25
All AJHSME/AMC 8 Problems and Solutions